Đến nội dung

Hình ảnh

Bạn đã tìm lời giải như thế nào ?

* * * * * 11 Bình chọn

  • Please log in to reply
Chủ đề này có 16 trả lời

#1
Nguyenhuyen_AG

Nguyenhuyen_AG

    Trung úy

  • Thành viên nổi bật 2016
  • 945 Bài viết

*
Phổ biến

Trước hết mình xin chúc các thành viên của Diễn Đàn Toán Học năm mới vui vẻ và hạnh phúc, đạt được nhiều thành công trong công việc cũng như học tập và ... tìm gấu  :lol:

 

Bây giờ mình sẽ vào vấn đề chính. Như chúng ta đã thấy đa phần những sách tham khảo hiện nay, đều viết theo kiểu thị trường "nêu phương pháp rồi giải" chứ không làm được một điều, đó là giải thích cho bạn đọc hiểu được cặn kẽ cội nguồn của ý tưởng: Quan sát bài toán thì thấy gì, định hướng ra làm sao, ta nên làm gì với nó, ...
 
Chính điều đó đã dẫn đến một hệ lụy là, khi gặp những "dạng mới" thì học sinh bị bỡ ngỡ (do suốt ngày chỉ học "mấy dạng" trong các sách kia thôi mà) và thu được những kết quả không tốt. Vì những nguyên nhân như vậy, nay mình xin mở topic này, nêu ra các ví dụ để mọi người cùng nhau thảo luận và phân tích cho bạn đọc biết thế nào là một cách học đúng (không máy móc). Vì vậy rất mong nhận được sự nhiệt tình từ phía các thành viên của diễn đàn.
 
Tất nhiên là không phải bài toán nào cũng có thể nêu ra được ý tưởng nó còn phụ thuộc vào nhiều ý tố mà đa phần là do kinh nghiệm của từng người. Chẳng hạn như nếu đưa ra bài toán sau
Cho $a,\;b,\;c$ là ba số thực tùy ý. Hãy chứng minh
\[(a^2+b^2+c^2)^2 \ge 3(a^3b+b^3c+c^3a),\]
mà yêu cầu chúng ta nêu ra ý tưởng để giải thì đúng là éo le vì thậm chí một lời giải tự nhiên cho bài toán này vẫn chưa được tìm ra thì làm sao có thể cho ý tưởng để giải được.
 
Chốt, topic này chúng ta không chú trọng việc giải! Mà chú trọng vào việc phân tích và bình luận. Và hy vọng rằng mọi người sẽ chia sẻ bằng tất cả lòng nhiệt huyết, không có tình trạng giấu nghề.

 

Mình xin bắt đầu bằng bài toán sau:
 

Bài toán 1. Với $a,\;b$ là hai số thực dương tùy ý. Hãy chứng minh bất đẳng thức 

\[\left( {{a}^{2}}+b+\frac{3}{4} \right)\left( {{b}^{2}}+a+\frac{3}{4} \right)\ge \left( 2a+\frac{1}{2} \right)\left( 2b+\frac{1}{2} \right).\]

Lời giải. Quan sát một chút ta thấy mỗi biểu thức trong ngoặc ở vế phải có thể phân tách thành tổng của hai số hạng mà một trong nó sẽ giống với một thừa số bên vế phải, cụ thể là

\[2\left( {{a}^{2}}+b+\frac{3}{4} \right)=\left( 2{{a}^{2}}+1 \right)+\left( 2b+\frac{1}{2} \right).\]

Mặt khác với $a=b=\frac{1}{2}$ thì đẳng thức của bài toán xảy ra, đồng thời $2{{a}^{2}}+1=2b+\frac{1}{2},$ vì thế nếu sử dụng đẳng thức này sau đó sử dụng tiếp bất đẳng thức AM-GM cho vế trái thì ta có thể giản ước bớt đi đại lượng $\left( 2a+\frac{1}{2} \right)\left( 2b+\frac{1}{2} \right)$ cho vế phải mà vẫn đảm bảo được dấu bằng của bài toán.

 

Ý tưởng là như vậy và ta sẽ tiến hành như sau. Viết bất đẳng thức bất đẳng thức cần chứng minh lại dưới dạng

\[\left[ \left( 2{{a}^{2}}+1 \right)+\left( 2b+\frac{1}{2} \right) \right]\left[ \left( 2{{b}^{2}}+1 \right)+\left( 2a+\frac{1}{2} \right) \right]\ge 4\left( 2a+\frac{1}{2} \right)\left( 2b+\frac{1}{2} \right).\]

Sử dụng bất đẳng thức AM-GM, ta có

$$\left( 2{{a}^{2}}+1 \right)+\left( 2b+\frac{1}{2} \right)\ge 2\sqrt{( 2{{a}^{2}}+1)\left( 2b+\frac{1}{2} \right)},$$

\[\left( 2{{b}^{2}}+1 \right)+\left( 2a+\frac{1}{2} \right)\ge 2\sqrt{\left( 2{{b}^{2}}+1 \right)\left( 2a+\frac{1}{2} \right)}.\]

Từ đánh giá trên ta đưa bài toán về chứng minh

\[\left( 2{{a}^{2}}+1 \right)\left( 2{{b}^{2}}+1 \right)\ge \left( 2a+\frac{1}{2} \right)\left( 2b+\frac{1}{2} \right).\]

Bất đẳng thức cuối cùng được suy ra từ đánh giá cơ bản sau đây

\[2{{x}^{2}}+1\ge 2x+\frac{1}{2},\]

với $x$ là số thực bất kỳ.

 

Nhưng điều này là hiển nhiên vì

\[2{{x}^{2}}+1-\left( 2x+\frac{1}{2} \right)=2{{\left( x-\frac{1}{2} \right)}^{2}}\ge 0. \]

Bài toán được chứng minh.

 

Trên đây là ý tưởng của mình để giải bài toán này. Nó khá dễ nên cách chứng minh của nó cũng dễ tìm ra, nhưng điều mình muốn nói ở đây là ý tưởng. Những ý tưởng trên có thể chưa phải là hay nhất, nhưng cũng từ những suy luận có lý mà ra. :D Các bạn hãy thử đề xuất bài toán (có thể nêu được ý tưởng giải) để mọi người cùng bàn luận nhé. 

 

À, không biết bài toán trên còn hướng tiếp cận nào khác không nhỉ ?


Bài viết đã được chỉnh sửa nội dung bởi Nguyenhuyen_AG: 01-01-2014 - 01:18

Nguyen Van Huyen
Ho Chi Minh City University Of Transport

#2
Ispectorgadget

Ispectorgadget

    Nothing

  • Quản lý Toán Phổ thông
  • 2946 Bài viết

Spoiler

Bài 2:  Cho các số thực không âm $a,b,c.$ Chứng minh

$$a^3+b^3+c^3-a^2b-b^2c-c^2a \ge 3(a-b)(b-c)(c-a)$$

 

 

Trước tiên ta thấy rằng đây là một bất đẳng thức hoán vị, vì vậy sẽ thuận lợi cho chúng ta trong việc sắp xếp lại các biến và chia trường hợp. Chúng ta hãy theo dõi lời giải sau mà kĩ thuật sắp xếp biến và chia trường hợp mang lại đặc  biệt trong những bất đẳng thức hoán vị.

 

Lời giải:

 

Trường hợp 1: Giả sử: $a\ge b\ge c$

 

Khi đó ta có: $$(a-b)(b-c)(c-a)\le 0$$

 

Khi đó ta chỉ cần chứng minh $$a^3+b^3+c^3 \ge a^2b+b^2c+c^2a$$

Thật vậy, áp đụng bất đẳng thức AM-GM ta có:

$$\frac{a^3+a^3+b^3}{3}\ge a^2b$$

$$\frac{b^3+b^3+c^3}{3}\ge b^2c$$

$$\frac{c^3+c^3+a^3}{3}\ge c^2a$$

Cộng 3 bất đẳng thức này lại ta có điều cần chứng minh.

 

Trường hợp 2: Khi $a\le b\le c$ khi đó $(a-b)(b-c)(c-a)\ge 0$

Bất đẳng thức cần chứng minh $$\Leftrightarrow a^2(a-b)+b^2(b-c)+c^2(c-a)\ge 3(a-b)(b-c)(c-a)$$

$$\Leftrightarrow (a-b)(a^2-3(b-c)(c-a))+(b-c)(b^2-3(a-b)(c-a))+(c-a)(c^2-3(a-b)(b-c))\ge 0$$

Đây là một bất đẳng thức đúng do $c\ge b\ge a$.

 

Vậy từ hai trường hợp trên ta có điều phải chứng minh. $\square$

 

Nhận xét: Với những bất đẳng thức hoán vị, ta có được một số lợi ích trong việc sắp xếp lại các biến và phân chia trường hợp như sau:

Khi $a\ge b\ge c$ ta được

 

$$(a-b)(b-c)(c-a)\le 0 \Leftrightarrow a^2b+b^2c+c^2a \ge ab^2+bc^2+ca^2$$

 

$$(a+b+c)(a-b)(b-c)(c-a)\le 0 \Leftrightarrow a^3b+b^3c+c^3a\ge ab^3+bc^3+ca^3$$

...

Kĩ thuật này đôi khi khá dài dòng và đòi hỏi tính toán nhưng nó giúp cho ta thêm điều kiện để chứng minh bất đẳng thức.

 

Với phương pháp làm tương tự ta có thể giải quyết các bài toán sau:

 

(VIF) Cho các số thực dương $a,b,c$. Chứng minh

$$\frac{a^2+b^2+c^2}{ab+bc+ac}+\frac{3}{2}.\frac{2a^2b+2b^2c+2c^2a-abc}{2ab^2+2bc^2+2ca^2-abc}\ge \frac{5}{2}$$

 

Cho các số thực không âm $a,b,c$. Chứng minh $$2(a+b+c)^3 \ge 9(a^2b+b^2c+c^2a)+3(a+b+c)(ab+bc+ac)$$


Bài viết đã được chỉnh sửa nội dung bởi Ispectorgadget: 01-01-2014 - 11:20

►|| The aim of life is self-development. To realize one's nature perfectly - that is what each of us is here for. ™ ♫


#3
Nguyenhuyen_AG

Nguyenhuyen_AG

    Trung úy

  • Thành viên nổi bật 2016
  • 945 Bài viết
 



Spoiler

Bài 2:  Cho các số thực không âm $a,b,c.$ Chứng minh

$$a^3+b^3+c^3-a^2b-b^2c-c^2a \ge 3(a-b)(b-c)(c-a)$$

 

 

Bài này mình giải có phần hơi thiên về kinh nghiệm một tí. Vì mình thấy những bài toán dạng đa thức như vậy thì kỹ thuật Schur - SOS là một giải pháp tương đối tốt. Tuy nhiên đối với bài toán này thì mình chỉ mô phỏng theo kỹ thuật Schur - SOS chú không hoàn toàn dùng nó.

 

Với nhận định đó, mình viết bất đẳng thức trên lại như sau

\[a^3+b^3+c^3+2(a^2b+b^2c+c^2a)\ge 3(ab^2+bc^2+ca^2),\]

hay là

\[(a^3+b^3+c^3-3abc)+2(a^2b+b^2c+c^2a-3abc)\ge 3(ab^2+bc^2+ca^2-3abc).\]

Ta có các phân tích sau

\[a^3+b^3+c^3-3abc=(a+b+c)(a-b)^2+(a+b+c)(a-c)(b-c).\]

\[a^2b+b^2c+c^2a-3abc=c(a-b)^2+a(a-c)(b-c).\]

\[ab^2+bc^2+ca^2-3abc=c(a-b)^2+b(a-c)(b-c).\]

Từ đó có thể viết bất đẳng thức cần chứng minh lại dưới dạng

$$(a+b)(a-b)^2+(3a-2b+c)(a-c)(b-c) \ge 0.$$

Lưu ý là từ đầu đến giờ chúng ta vẫn chưa sắp thứ tự các biến hoặc chọn phần tử cực hạn. Nên chỉ có là đại lượng $(a+b)(a-b)^2$ là hiển nhiên không âm, nên để chứng minh được bài toán thì ta cần có $(3a-2b+c)(a-c)(b-c)$ cũng không âm.

 

Ta thấy, chỉ cần $a \ge b$ còn $c$ tùy ý là số lớn nhất hay nhỏ nhất thì bài toán được chứng minh.Tuy nhiên ta không thể giả sử được điều này vì bất đẳng thức đã cho chỉ có tính hoán vị chứ không đối xứng. Nên ý tưởng giả sử $c$ là số nhỏ nhất hay số lớn nhất trong hướng đi này hơi khó cho chúng ta. Như vậy còn trường hợp $c$ là số ở giữa hai số còn lại thì sao ? 

 

Nếu ta tách $a-b=(a-c)+(c-b)$ và sau đó áp dụng bất đẳng thức AM-GM sẽ ta sẽ tạo ra được $(a-c)(c-b)$ sẽ có thể giản ước cho đại lượng $(a-c)(b-c)$ và đưa bài toán ba biến bậc 3 về thành ba biến bậc nhất.

 

Với ý tưởng này, ta giả sử $c$ là số nằm giữa $a$ và $b.$ Khi đó theo bất đẳng thức AM-GM, ta có

$$(a-b)=[(a-c)+(c-b)]^2 \ge 4(a-c)(c-b)\ge 0.$$

Vậy, ta cần chứng minh

\[4(a+b)(a-c)(c-b)+(3a-2b+c)(a-c)(b-c)\ge0.\]

hay là

\[(a-c)(c-b)(a+6b-c) \ge 0.\]

Do $(a-c)(c-b)\ge0$ nên ta chỉ cần chứng minh $a+6b-c\ge0.$ Ta thấy bất đẳng thức này tương đương với

\[(a-c)+6b\ge0,\]

\[(b-c)+a+5b \ge 0.\]

Vì $c$ là số ở giữa $a$ và $b$ nên một trong hai bất đẳng thức trên đúng. Ta có điều phải chứng minh

 

Nhận xét: Thông thường khi gặp các bài toán hoán vị thì ta thường đưa chúng về dạng đối xứng vì bất đẳng thức đối xứng thường dễ xử hơn bất đẳng thức hoán vị. Liệu ý tưởng này có áp dụng được cho bài toán của bạn Ispectorgadget không ?


Bài viết đã được chỉnh sửa nội dung bởi Ispectorgadget: 01-01-2014 - 17:26
Latex Fixed

Nguyen Van Huyen
Ho Chi Minh City University Of Transport

#4
Nguyenhuyen_AG

Nguyenhuyen_AG

    Trung úy

  • Thành viên nổi bật 2016
  • 945 Bài viết

Mọi người tiếp tục thảo luận nhé. Lúc nãy qua bên Mathscope, thấy có một bạn post bài này.

 

Bài 3. Cho $a,\;b,\;c$ là các số thực thay đổi tùy ý thỏa mãn $ab+bc+ca=1.$ Hãy tìm giá trị nhỏ nhất của biểu thức sau

\[P=a^2+2b^2+5c^2.\]


Bài viết đã được chỉnh sửa nội dung bởi Nguyenhuyen_AG: 01-01-2014 - 22:04

Nguyen Van Huyen
Ho Chi Minh City University Of Transport

#5
shinichikudo201

shinichikudo201

    Thiếu úy

  • Thành viên
  • 521 Bài viết

Mình xin đóng góp bài này:

Bài toán 4. Cho các số thực dương a,b,c thỏa mãn:

$\frac{1}{1+a}+\frac{1}{1+b}+\frac{1}{1+c}\geq 2$

Chứng minh $xyz\leq \frac{1}{8}$

Lời giải

Ta để ý thấy rằng $\frac{1}{1+x}= \frac{x+1-x}{1+x}=1-\frac{x}{1+x}$. Chính vì vậy nên ta đánh giá giả thiết như sau:

$\frac{1}{1+a}+\frac{1}{1+b}+\frac{1}{1+c}\geq 2$

$\Leftrightarrow \frac{1}{1+x}\geq (1-\frac{1}{1+y})+(1-\frac{1}{1+z})$

$\Leftrightarrow \frac{1}{1+x}\geq \frac{y}{1+y}+\frac{z}{1+z}$

Đến đây thấy bên vế trái có (1+x) ở mẫu, vế phải có (1+y) và (1+z) ở mẫu. Ta lại thấy nếu tiếp tục đánh giá như vậy thì hai BĐT với hai số hạng còn lại $\frac{1}{1+y}$ và $\frac{1}{1+z}$ cũng có dạng tương tự. Mặt khác các tử ở các vế phải có xuất hiện x,y,z. Từ đó ta có ý tưởng chuyển các số hạng ở vế phải thành tích rồi nhân vế với vế các bất đẳng thức cùng chiều là có đpcm. Điều này được thực hiện bằng bất đẳng thức AM-GM:

$\frac{y}{1+y}+\frac{z}{1+z}$$\geq$$2\sqrt{\frac{yz}{(1+y)(1+z)}}$

 

$\Rightarrow \frac{1}{1+x}\geq 2\sqrt{\frac{yz}{(1+y)(1+z)}}$

Chứng minh tương tự rồi nhân các BĐT cùng chiều:

$\frac{1}{1+x}.\frac{1}{1+y}.\frac{1}{1+z}\geq 2^{3}.\sqrt{\frac{x^{2}y^{2}z^{2}}{(1+x)(1+y)(1+z)^{2}}}$

 

$\Leftrightarrow 8xyz\leq 1\Leftrightarrow xyz\leq \frac{1}{8}$ (đpcm)

Đẳng thức xảy ra $\Leftrightarrow$ $x=y=z=\frac{1}{2}$

Mọi người góp ý  :mellow:  :mellow:  :mellow:


Bài viết đã được chỉnh sửa nội dung bởi henry0905: 17-01-2014 - 23:04
Đánh số bài.

It is the quality of one's convictions that determines successnot the number of followers


#6
Nguyenhuyen_AG

Nguyenhuyen_AG

    Trung úy

  • Thành viên nổi bật 2016
  • 945 Bài viết

$\Leftrightarrow \frac{1}{1+x}\geq \frac{y}{1+y}+\frac{z}{1+z}$

 

 

Sao bạn lại chuyển hai đại lương kia qua bên vế trái để viết điều kiện của bài toán lại như vậy ? Đoạn này mình vẫn chưa hiểu lắm. :v


Nguyen Van Huyen
Ho Chi Minh City University Of Transport

#7
Nguyenhuyen_AG

Nguyenhuyen_AG

    Trung úy

  • Thành viên nổi bật 2016
  • 945 Bài viết


Mọi người tiếp tục thảo luận nhé. Lúc nãy qua bên Mathscope, thấy có một bạn post bài này.

 

Bài 3. Cho $a,\;b,\;c$ là các số thực thay đổi tùy ý thỏa mãn $ab+bc+ca=1.$ Hãy tìm giá trị nhỏ nhất của biểu thức sau

\[P=a^2+2b^2+5c^2.\]

 

Mấy tuần này mình bận thi học kỳ nên không vào diễn đàn thường xuyên được. Giờ mình sẽ giải bất đẳng thức này, chúng ta sẽ giải quyết bài toán tổng quát luôn

 

Với $a,\;b$ là hai số thực dương cho trước và $x,\;y,\;z$ là các số thực thay đổi thỏa mãn điều kiện $xy+yz+zx=1.$
 
$(a)$ Chứng minh rằng phương trình.
$$2t^3+(1+a+b)t^2-ab=0,$$
có nghiệm dương duy nhất.
 
$(b)$ Tìm giá trị nhỏ nhất của biểu thức
$$P=x^2+ay^2+bz^2. \quad (5)$$
Phần $(a)$ khá đơn giản, có thể tự chứng mình lấy bằng cách sử dụng đạo hàm. Ở đây ta chỉ quan tâm đến phần $(b)$ của bài toán.
 
Ta thấy biểu thức $P$ của bài toán có dạng $A^2+B^2+C^2,$ rất giống với bất đẳng thức Cauchy-Schwaz dạng phân thức là
$$\frac{A^2}{u}+\frac{B^2}{v}+\frac{C^2}{w} \ge \frac{(A+B+C)^2}{u+v+w}.$$
Điều này gợi ý cho ta sử dụng bất đẳng thức Cauchy-Schwarz để chứng minh bài toán.
 
Tuy nhiên, vế trái của ta đã có dạng tổng các bình phương rồi, nhưng biểu thức bình phương ở vế phải sẽ như thế nào ? Bây giờ ta sẽ tìm cách tạo ra bình phương ở vế phải bằng cách sử dụng giả thiết $xy+yz+zx=1.$
 
Trước hết, ta sẽ thuần nhất bài toán để đưa bài toán về đúng với bản chất của nó. Giả sử $P$ đạt giá trị nhỏ nhất bằng $2t \;(t>0)$ (tại sao là $2t$ mà không phải là số nào khác ? t chẳng hạn) Khi đó ta có bất đẳng thức sau
$$x^2+ay^2+bz^2 \ge 2t.$$
Hay viết dưới dạng thuần nhất là 
$$x^2+ay^2+bz^2 \ge 2t(xy+yz+zx). \quad (5.1)$$
Đến đây thì ý tưởng tạo bình phương bên vế phải đã xuất hiện, vì chỉ cần thêm vào đại lượng $t(x^2+y^2+z^2)$ ta sẽ có ngay $2t(xy+yz+zx)+t(x^2+y^2+z^2)=t(x+y+z)^2.$ Đến đây chắc hẳn ta đã thấy được tác dụng của việc chọn giá trị nhỏ nhất của $P$ là $2t$ thay vì là một số khác. Như là $t$ chẳng hạn, vì khi đó ta phải cộng thêm vào một đại lượng $\frac{t(x^2+y^2+z^2)}{2}$ là một phân thức thay vì dạng đơn giản hơn $t(x^2+y^2+z^2).$
 
Với ý tương trên, ta có thế viết $(5.1)$ lại như sau
$$x^2+ay^2+bz^2+t(x^2+y^2+z^2) \ge 2t(xy+yz+zx)+t(x^2+y^2+z^2),$$
tương đương với
$$(t+1)x^2+(t+a)y^2+(t+b)z^2\ge t(x+y+z)^2.$$
Sử dụng bất đẳng thức Cauchy-Schwarz, ta có 
$$(t+1)x^2+(t+a)y^2+(t+b)z^2 \ge \frac{(x+y+z)^2}{\frac{1}{t+1}+\frac{1}{t+a}+\frac{1}{t+b}}.$$
Ta cần cần chọn $t$ sao cho
$$\frac{(x+y+z)^2}{\frac{1}{t+1}+\frac{1}{t+a}+\frac{1}{t+b}}=\frac{(x+y+z)^2}{\frac{1}{t}},$$
hay là
$$\frac{1}{t+1}+\frac{1}{t+a}+\frac{1}{t+b}=\frac{1}{t},$$
quy đồng và thu gọn, ta được
$$2t^3+(1+a+b)t^2-ab=0.$$
Kết hợp với câu $(a)$ ta suy ra $P$ đạt giá trị nhỏ nhất là $2t$ với $t$ là nghiệm dương của đa thức
$$f(t)=2t^3+(1+a+b)t^2-ab.$$
Đẳng thức xảy ra khi và chỉ khi $(t+1)x=(t+a)y=(t+b)z.$

Nguyen Van Huyen
Ho Chi Minh City University Of Transport

#8
Ispectorgadget

Ispectorgadget

    Nothing

  • Quản lý Toán Phổ thông
  • 2946 Bài viết

$\boxed{\text{Bài 5.}}$ Cho $a,b,c\in [-1;1]$ thỏa $a+b+c=0$ chứng minh: $$a^{2012}+b^{2013}+c^{2014}\le 2$$
 
Phân tích: Từ giả thiết ta có $|a|\le 1;|b|\le 2;|c| \le 3$ và $a^{2012}\le |a|;b^{2013}\le |b|; c^{2014}\le |c|$.
 
Nên $$a^{2012}+b^{2013}+c^{2014} \le |a|+|b|+|c|$$
Từ giả thiết ta có $c=-a-b$, do đó bài toán sẽ kết thúc nếu ta đưa về được 1 biến nhưng ở đây bị vướng dấu trị tuyệt đối nên ta sẽ tìm cách biến đổi $|a|+|b|$ thành $|a+b|$ .
Ta có bất đẳng thức $|a|+|b| \ge |a+b|$ nhưng dùng bất đẳng thức này ta sẽ không được điều mong muốn >:) nhưng để ý 1 tí, dấu "=" của bất đẳng thức này là $a,b$ cùng dấu lợi dụng điều này ta sẽ có lời giải khá đẹp sau
 
Lời giải:
 
Từ giả thiết ta có: $a^{2012}\le |a|;b^{2013}\le |b|; c^{2014}\le |c|$
Nên $$a^{2012}+b^{2013}+c^{2014} \le |a|+|b|+|c|$$
 
Theo nguyên tắc Dirichlet trong 3 số $a,b,c$ ta luôn có ít nhất $2$ số cùng dấu, giả sử đó là $a$ và $b$.
 
Ta có $a^{2012}+b^{2013}+c^{2014}\le |a|+|b|+|c| \le |a+b|+|-c|=2|c|\le 2$
 
Vậy bài toán được chứng minh. $\square$
 
Tổng quát: Cho $a,b,c\in [-1;1]$ thỏa $a+b+c=0$. Chứng minh $$a^m+b^n+c^p\le 2$$
(Trong đó $m,n,p \in \mathbb{N}$ và lớn hơn hoặc bằng 2).

 

Spoiler


Bài viết đã được chỉnh sửa nội dung bởi Ispectorgadget: 17-01-2014 - 10:38

►|| The aim of life is self-development. To realize one's nature perfectly - that is what each of us is here for. ™ ♫


#9
shinichikudo201

shinichikudo201

    Thiếu úy

  • Thành viên
  • 521 Bài viết

Sao bạn lại chuyển hai đại lương kia qua bên vế trái để viết điều kiện của bài toán lại như vậy ? Đoạn này mình vẫn chưa hiểu lắm. :v

Mình phân tích cái này ở đoạn trên còn gì (cái này gần tương tự Cauchy ngược dấu)


It is the quality of one's convictions that determines successnot the number of followers


#10
Nguyenhuyen_AG

Nguyenhuyen_AG

    Trung úy

  • Thành viên nổi bật 2016
  • 945 Bài viết

Tình cờ thấy được bài này trên diễn đàn mọi người cùng phân tích thử xem nhé. 

 

$\boxed{\text{Bài 6.}}$ Cho hai số dương $a,\;b$ thỏa mãn điều kiện $a^2+b^2=5.$ Hãy tìm giá trị nhỏ nhất của biểu thức

\[P=a^3+b^6.\]


Nguyen Van Huyen
Ho Chi Minh City University Of Transport

#11
Nguyenhuyen_AG

Nguyenhuyen_AG

    Trung úy

  • Thành viên nổi bật 2016
  • 945 Bài viết

Tình cờ thấy được bài này trên diễn đàn mọi người cùng phân tích thử xem nhé. 

 

$\boxed{\text{Bài 6.}}$ Cho hai số dương $a,\;b$ thỏa mãn điều kiện $a^2+b^2=5.$ Hãy tìm giá trị nhỏ nhất của biểu thức

\[P=a^3+b^6.\]

 

Bài này có ý tưởng AM-GM rất rõ ràng, giả thiết của bài toán chứa biểu thức có bậc nhỏ hơn bậc của $P,$ và việc sử dụng bất đẳng thức AM-GM để hạ bậc là một ý tưởng rất tự nhiên. Tuy nhiên, chúng ta lại không biết là dấu bằng của bài toán sẽ xảy ra khi nào nên việc chọn hệ số để sử dụng bất đẳng thức AM-GM cũng khá vất vả, vì vậy ta sẽ sử dụng kỹ thuật chọn dấu bằng giả định.

 

Cụ thể, ta sẽ giả sử đẳng thức của bài toán sẽ đạt được tại $a=x,\;b=y$ với $x,\;y$ là những hằng số. Khi đó theo bất đẳng thức AM-GM, ta có

\[a^3+a^3+x^3 \ge 3\sqrt[3]{a^3\cdot a^3 \cdot x^3}=3a^2\cdot x, \quad (1)\]

tức là

\[2a^3+x^3 \ge 3a^2\cdot x,\]

\[a^3\ge \frac{3a^2\cdot x-x^3 }{2}.\]

Hoàn toàn tườn tự, ta có 

\[b^6\ge 3b^2\cdot y^4-2y^6\]

Như vậy, ta có 

\[ a^3+b^6 \ge 3b^2\cdot y^4-2y^6+\frac{3a^2\cdot x}{2}-\frac{x^3}{2}=3\left ( b^2\cdot y^2+a^2\cdot \frac{x}{2} \right )-2y^6-\frac{x^3}{2}. \quad (2)\]

Ta cần chọn $x,\;y$ sao cho trong $(2)$ ta có thể sử sụng được giả thiết $a^2+b^2=5$ đồng thời cũng thoả mãn điều kiện của bait tóan, tức $x,\;y$ phải thoả mãn hệ  phương trình

\[\left\{\begin{matrix}x^2+y^2&=5 \\y^2&=\frac{x}{2}\end{matrix}\right.\]

Giải hệ này ta được $x=2,\;y=1$ và suy ra $P_{\min} =9.$

 

Nhận xét: Có một số bạn sẽ đặt câu hỏi rằng tại sao lại dùng $2$ đại lượng $a^3$ kết hợp với $x^3$ để sử dụng bất đẳng thức AM-GM cho ba số. Để ý rằng điều kiện của bài toán cho ta $a^2+b^2=5$ nên chúng ta phải cố gắng đánh giá $a^3$ sao cho xuất hiện được $a^2$ và $b^6$ sao cho xuất hiện được $b^2$ để áp dụng giả thiết $a^2+b^2=5.$ Điều này lý giải cho việc áp dụng bất đẳng thức AM-GM như trên.


Bài viết đã được chỉnh sửa nội dung bởi Nguyenhuyen_AG: 01-03-2014 - 11:53

Nguyen Van Huyen
Ho Chi Minh City University Of Transport

#12
Ispectorgadget

Ispectorgadget

    Nothing

  • Quản lý Toán Phổ thông
  • 2946 Bài viết

$\boxed{\text{Bài 6.}}$ [/size]Cho hai số dương $a,\;b$ thỏa mãn điều kiện $a^2+b^2=5.$ Hãy tìm giá trị nhỏ nhất của biểu thức
\[P=a^3+b^6.\]

>:) Dùng cách tà đạo tí: Phương pháp nhân tử Lagrange
Như phân tích ở trên mấu chốt bài toán này là tìm điểm rơi của bài toán.
Ta thiết lập hàm Lagrange như sau
$$f(a;b)=a^3+b^6-\lambda(a^2+b^2-5)$$
Điểm cực trị là nghiệm của hệ
\[\left\{ \begin{array}{l}
\frac{{\partial f}}{{\partial a}} = \frac{{\partial f}}{{\partial b}} = \frac{{\partial f}}{{\partial c}} = 0\\
{a^2} + {b^2} = 5
\end{array} \right.\]
 
$\frac{\partial f}{\partial a}=3a^2-2\lambda a=0 \Leftrightarrow \lambda =\frac{3a}{2}$

$\frac{\partial f}{\partial b}=6b^5-2b\lambda \Rightarrow \lambda =3b^4$
 
Từ đó Ta có $a=2b^4$
 
Ta giải hệ sau bằng phép thế \[\left\{ \begin{array}{l}
{a^2} + {b^2} = 5\\
a = 2{b^4}
\end{array} \right.\]
Ta được $a=2;b=1$
( >:) Có thể dùng chức năng Shilf + Solve trong máy tính bỏ túi để tìm)
Từ đây ta trình bày lời giải như bài viết ở trên.
 
Tương tự ta có bài toán sau:
Cho $a,b,c$ thực dương thỏa $2a+2b+c^2=14$. Tìm giá trị nhỏ nhất của $$P=a^3+2b^2+2c^3$$ (Trích đề thi thử Moon.vn lần 2-2014)
 

Spoiler


►|| The aim of life is self-development. To realize one's nature perfectly - that is what each of us is here for. ™ ♫


#13
Nguyenhuyen_AG

Nguyenhuyen_AG

    Trung úy

  • Thành viên nổi bật 2016
  • 945 Bài viết

 Tương tự ta có bài toán sau:
Cho $a,b,c$ thực dương thỏa $2a+2b+c^2=14$. Tìm giá trị nhỏ nhất của $$P=a^3+2b^2+2c^3$$ (Trích đề thi thử Moon.vn lần 2-2014)
 

Spoiler

 

Hoàn toàn tương tự. Ta giả sử $a=x,\;b=y,\;c=z$ và sử dụng bất đẳng thức AM-GM để tạo ra $a,\;b$ và $b^2.$ Ta có

\[a^3+2b^2+2c^3 \ge \left ( 3ax^2+4by+3c^2z \right )-\left ( 2x^3+2y^2+z^3 \right ).\]

Để sử dụng được điều kiện thì ta cần tách

\[3ax^2+4by+3c^2z=2a\cdot \frac{3x^2}{2}+2b\cdot2y+c^2\cdot3z.\]

Như vậy ta cần chọn $x,\;y,\;z$ thoả mãn điều kiện

\[\left\{\begin{matrix}&\dfrac{3x^2}{2}=2y=3z \\& 2x+2y+z^2=14\end{matrix}\right.\] 

Giải hệ này ta được $x=2,\;y=3,\;z=2$ và có được $P_{\min} = 34.$


Nguyen Van Huyen
Ho Chi Minh City University Of Transport

#14
Nguyenhuyen_AG

Nguyenhuyen_AG

    Trung úy

  • Thành viên nổi bật 2016
  • 945 Bài viết

Mọi người tiếp tục nhé. Đây là bài thi quốc gia năm 2008.

 

$\boxed{\text{Bài 7.}}$ Với $x,\,y,\,z$ là ba số thực không âm đôi một khác nhau, hãy chứng minh

\[(xy+yz+zx)\left [\frac{1}{(x-y)^2}+\frac{1}{(y-z)^2}+\frac{1}{(z-x)^2} \right ] \ge 4.\]


Nguyen Van Huyen
Ho Chi Minh City University Of Transport

#15
Ispectorgadget

Ispectorgadget

    Nothing

  • Quản lý Toán Phổ thông
  • 2946 Bài viết

Mọi người tiếp tục nhé. Đây là bài thi quốc gia năm 2008.

 

$\boxed{\text{Bài 7.}}$ Với $x,\,y,\,z$ là ba số thực không âm đôi một khác nhau, hãy chứng minh

\[(xy+yz+zx)\left [\frac{1}{(x-y)^2}+\frac{1}{(y-z)^2}+\frac{1}{(z-x)^2} \right ] \ge 4.\]

Với ý tưởng dồn biến toàn miền quen thuộc :
Không mất tính tổng quát, giả sử $z=\min\{x;y;z\}$. Ta giảm $(x,y,z)$ cùng đi một lượng $z$ (tức là thay $(x;y;z)$ bởi $(x-z;y-z;0)$) thì rõ ràng các hiệu $x-y;y-z;z-x$ không đổi , và $(x-z)(y-z)<xy+yz+zx$. Vậy $VT(*)$ thì giảm đi còn $VP(*)$ thì không đổi. Do đó ta chỉ cần c/m BĐT $(*)$ trong trường hợp $x,y>z=0$.

Thật vậy, với $x,y>z=0$ thì:
$$(*) \Leftrightarrow xy \left [ \dfrac{1}{(x-y)^2}+\dfrac{1}{x^2}+\dfrac{1}{y^2} \right ]\ge 4\\ \Leftrightarrow \dfrac{xy}{(x-y)^2}+\dfrac{x^2+y^2}{xy}\ge 4\\ \Leftrightarrow \dfrac{xy}{(x-y)^2}+\dfrac{(x-y)^2}{xy}\ge 2$$

Hiển nhiên đúng theo $AM-GM$. BĐT $(*)$ được chứng minh.  Dấu bằng xảy ra khi:$\left\{\begin{matrix}(x-y)^2=xy\\ z=0\end{matrix}\right.$

$\Leftrightarrow (x;y;z)=\left (k;\dfrac{k(3+\sqrt{5})}{2};0\right );\left (k;\dfrac{k(3-\sqrt{5})}{2};0\right )\ (k> 0)$. và các hoán vị $\square$

Spoiler


►|| The aim of life is self-development. To realize one's nature perfectly - that is what each of us is here for. ™ ♫


#16
hoilamchi

hoilamchi

    Trung sĩ

  • Thành viên
  • 164 Bài viết

Cho em hỏi về phương pháp chuẩn hoá trong Bất đẳng thức được không ạ?Khi nào thì được sử dụng và cách sử dụng?



#17
hieutoan

hieutoan

    Binh nhất

  • Thành viên
  • 23 Bài viết

cho a,b,c $\geq$0. CMR a(a-b)(a-c)+b(b-c)(b-a)+c(c-a)(c-b) $\geq \sum ab(\sqrt{a}-\sqrt{b})^{2}$






2 người đang xem chủ đề

0 thành viên, 2 khách, 0 thành viên ẩn danh